Prove x^{k+1}-1=(x-1)(x^{k}+x^{k-1}+...+1) by Induction

  • Thread starter dtl42
  • Start date
  • Tags
    Induction
In summary, the homework statement is proving that x^{k}-1=(x-1)(x^{k-1}+x^{k-2}+...+1) by induction. The Attempt at a Solution tried substituting in x^{k} term and expanding x*x^{k}-1, but it failed. The Homework Equations state that x^{k+1}-1=(x-1)(x^{k}+x^{k-1}+...+1) and I need to prove this by assuming that x^{k}-1=(x-1)(x^{k-1}+x^{k-2}+...+1). The Attempt at a Solution found
  • #1
dtl42
119
0

Homework Statement


Prove by Induction that [tex]x^{k}-1=(x-1)(x^{k-1}+x^{k-2}+...+1)[/tex]


Homework Equations


None? I need to prove [tex]x^{k+1}-1=(x-1)(x^{k}+x^{k-1}+...+1)[/tex] by assuming this is true: [tex]x^{k}-1=(x-1)(x^{k-1}+x^{k-2}+...+1)[/tex]


The Attempt at a Solution


[tex]x^{k+1}-1=(x-1)(x^{k}+x^{k-1}+...+1)[/tex] - I tried substituting in the [tex]x^{k}[/tex] term, and expanding [tex]x*x^{k}-1[/tex], but it failed.
 
Physics news on Phys.org
  • #2
dtl42 said:

Homework Statement


Prove by Induction that [tex]x^{k}-1=(x-1)(x^{k-1}+x^{k-2}+...+1)[/tex]


Homework Equations


None? I need to prove [tex]x^{k+1}-1=(x-1)(x^{k}+x^{k-1}+...+1)[/tex] by assuming this is true: [tex]x^{k}-1=(x-1)(x^{k-1}+x^{k-2}+...+1)[/tex]


The Attempt at a Solution


[tex]x^{k+1}-1=(x-1)(x^{k}+x^{k-1}+...+1)[/tex]
You can't use that- that's what you are trying to prove.

- I tried substituting in the [tex]x^{k}[/tex] term, and expanding [tex]x*x^{k}-1[/tex], but it failed.

You have xk+1- 1 and you need to use the fact that xk- 1= (x-1)(xk-1+...+ x+ 1). Have you tried dividing xk+1-1 by xk- 1? It does not divide evenly, you get
[tex]x^{k+1}-1= (x^k-1)(x+1)+ \frac{x-1}{x^k-1}[/tex]
but now you should be able to simplify that fraction.
 
  • #3
That fraction would simplify to [tex]\frac{1}{x^{k}+x^{k-1}+...+1}[/tex] right?
 
  • #4
That fraction would simplify to [tex]\frac{1}{x^{k}+x^{k-1}+...+1}[/tex] right?
 
  • #5
Bump, I could use some help
 
  • #6
Check the power of the leading term in the denominator again.
 
  • #7
there is another method without any division. When you have [tex]x^{k+1}-1=x \cdot x^k-1[/tex], substitute the x part inte another appropriate term that equals x, then use the induction assumption.
 
Last edited:
  • #8
What would the other appropriate term be? I can't really work that out in my head, I thought I should substitute for [tex] x^{k} [/tex] because I can separate it more easily.
 
  • #9
I can give you an example of what technique I mean.
Prove that [tex]3^{2n+1} + 4^{2n+1}[/tex] always is divisible by 7.
Indcution. True for n=0, now assume true for n and consider n+1:
[tex]a_{n+1}=3^{2(n+1)+1} + 4^{2(n+1)+1}=9\cdot 3^{2n+1}+16 \cdot 4^{2n+1}=[/tex]
[tex]=9\cdot 3^{2n+1}+(9+7) \cdot 4^{2n+1}=9 \cdot a_n + 7 \cdot 4^{2n+1}[/tex]
which is divisible by 7 if a_n is.
How can you apply this method on your problem? how can the two terms look like, if they should equal x? :)
 
Last edited:

1. What is the purpose of using induction to prove this equation?

The purpose of using induction in this proof is to show that the equation holds for all values of k, not just for a specific value. This allows us to prove the equation for an infinite number of cases.

2. How does the induction proof work?

The induction proof works by first showing that the equation holds for a base case, typically when k=1. Then, assuming that the equation holds for a certain value of k, we can use that assumption to prove that the equation also holds for k+1. This process is repeated until we can conclude that the equation holds for all values of k.

3. What is the base case in this induction proof?

The base case in this proof is when k=1. This is because the equation is relatively simple to prove for this value and serves as the starting point for the rest of the proof.

4. How do you prove the inductive step in this proof?

To prove the inductive step, we assume that the equation holds for a certain value of k and use that assumption to prove that it also holds for k+1. This is typically done by substituting k+1 into the equation and simplifying until it matches the equation for the next value of k.

5. What are some common mistakes to avoid in this proof?

Some common mistakes to avoid in this proof include not properly stating the base case, not clearly showing the inductive step, or assuming that the equation holds for all values of k without proving it. It's important to be precise and thorough in each step of the proof to avoid any errors.

Similar threads

  • Precalculus Mathematics Homework Help
Replies
4
Views
653
  • Precalculus Mathematics Homework Help
Replies
6
Views
688
  • Precalculus Mathematics Homework Help
Replies
13
Views
1K
  • Precalculus Mathematics Homework Help
Replies
1
Views
1K
  • Precalculus Mathematics Homework Help
Replies
6
Views
1K
  • Precalculus Mathematics Homework Help
Replies
10
Views
791
  • Precalculus Mathematics Homework Help
Replies
2
Views
247
  • Precalculus Mathematics Homework Help
Replies
7
Views
1K
Replies
2
Views
2K
  • Precalculus Mathematics Homework Help
Replies
3
Views
631
Back
Top